Difference between revisions of "2003 AIME II Problems/Problem 9"

m
Line 3: Line 3:
  
 
== Solution ==
 
== Solution ==
{{solution}}
+
<math>{{Q(z_1)=0</math> therefore
 +
<math>z_1^4-z_1^3-z_1^2-1=0</math>
 +
therefore <math>-z_1^3-z^2=-z_1^4+1.</math>
 +
Also  <math>z_1^4-z_1^3-z_1^2=1 </math>
 +
 
 +
S0    <math>z_1^6-z_1^5-z_1^4=z_1^2</math>
 +
 
 +
So in <math>P(z_1)=z_1^6-z_1^5-z_1^3-z_1^2-z_1</math>
 +
      <math>P(z_1)=z_1^6-z_1^5-z_1^4-z_1+1</math>
 +
      <math>P(z_1)=z_1^2-z_1+1</math>
 +
Now this also follows for all roots of <math>Q(x)</math>
 +
Now <math>P(z_2)+P(z_1)+z_3)+P(z_4)=z_1^2-z_1+1+z_2^2-z_2+1+z_3^2-z_3+1+z_4^2-z_4+1</math>
 +
 
 +
Now by Vieta's we know that <math>-z_4-z_3-z_2-z_1=-1</math>
 +
So by Newton Sums we can find <math>z_1^2+z_2^2+z_3^2+z_4^2</math>
 +
 
 +
<math>a_ns_2+a_n-1s_1+2a_n-1=0</math>
 +
 
 +
<math>(1)(s_2)+(-1)(1)+2(-1)=0</math>
 +
 
 +
<math>s_2-1-2=0</math>
 +
 
 +
<math>s_2=3</math>
 +
 
 +
So finally
 +
<math>P(z_2)+P(z_1)+z_3)+P(z_4)=3+4-1=\box{6}</math>
 +
 +
 
 +
}}
  
 
== See also ==
 
== See also ==
 
{{AIME box|year=2003|n=II|num-b=8|num-a=10}}
 
{{AIME box|year=2003|n=II|num-b=8|num-a=10}}

Revision as of 00:42, 14 January 2008

Problem

Consider the polynomials $P(x) = x^{6} - x^{5} - x^{3} - x^{2} - x$ and $Q(x) = x^{4} - x^{3} - x^{2} - 1.$ Given that $z_{1},z_{2},z_{3},$ and $z_{4}$ are the roots of $Q(x) = 0,$ find $P(z_{1}) + P(z_{2}) + P(z_{3}) + P(z_{4}).$

Solution

${{Q(z_1)=0$ (Error compiling LaTeX. Unknown error_msg) therefore $z_1^4-z_1^3-z_1^2-1=0$ therefore $-z_1^3-z^2=-z_1^4+1.$ Also $z_1^4-z_1^3-z_1^2=1$

S0 $z_1^6-z_1^5-z_1^4=z_1^2$

So in $P(z_1)=z_1^6-z_1^5-z_1^3-z_1^2-z_1$

     $P(z_1)=z_1^6-z_1^5-z_1^4-z_1+1$
     $P(z_1)=z_1^2-z_1+1$ 

Now this also follows for all roots of $Q(x)$ Now $P(z_2)+P(z_1)+z_3)+P(z_4)=z_1^2-z_1+1+z_2^2-z_2+1+z_3^2-z_3+1+z_4^2-z_4+1$

Now by Vieta's we know that $-z_4-z_3-z_2-z_1=-1$ So by Newton Sums we can find $z_1^2+z_2^2+z_3^2+z_4^2$

$a_ns_2+a_n-1s_1+2a_n-1=0$

$(1)(s_2)+(-1)(1)+2(-1)=0$

$s_2-1-2=0$

$s_2=3$

So finally $P(z_2)+P(z_1)+z_3)+P(z_4)=3+4-1=\box{6}$ (Error compiling LaTeX. Unknown error_msg)


}}

See also

2003 AIME II (ProblemsAnswer KeyResources)
Preceded by
Problem 8
Followed by
Problem 10
1 2 3 4 5 6 7 8 9 10 11 12 13 14 15
All AIME Problems and Solutions